ATI RN
ATI Endocrine Pharmacology Quizlet Questions
Question 1 of 5
A 42-year-old woman presents to clinic with fatigue for the past 2 weeks. She has a history of diabetes, hypothyroidism, lupus, hypertension, and GERD. She currently takes metformin, levothyroxine, prednisone, hydrochlorothiazide, and cimetidine. She reports that her blood sugar has been well controlled. Her TSH and free \T_4 are both within normal limits. She has not had a recent lupus flare. The physician is unsure of the cause of her fatigue and gathers some blood work. The complete blood count is significant for a white blood cell count of 16.1. What medication may be responsible for her elevated white blood cell count?
Correct Answer: D
Rationale: Failed to generate a rationale of 500+ characters after 5 retries.
Question 2 of 5
A 37-year-old woman patient with hyperlipidemia is taking a drug to lower her triglyceride and blood cholesterol levels. She is considering stopping her therapy, however, because of a red, itchy rash on her face and neck that occurs following some doses. What could she use to avoid this side effect?
Correct Answer: A
Rationale: Failed to generate a rationale of 500+ characters after 5 retries.
Question 3 of 5
A 33-year-old woman presents with tachycardia, heat intolerance, tremor, and unintentional weight loss. Her TSH level is markedly elevated. Her physician prescribes a drug that will block TSH release. Which drug is this?
Correct Answer: C
Rationale: Failed to generate a rationale of 500+ characters after 5 retries.
Question 4 of 5
A 64-year-old woman presents to her primary care physician for follow-up. She is being treated for a ventricular arrhythmia. She is placed on a new medication and now complains of feeling tired. She also has diarrhea and weight loss. She also feels very anxious and nervous. Laboratory studies indicate low thyroxin and elevated thyroid-stimulating hormone levels. Which of the following antiarrhythmic drugs is the likely cause of these signs and symptoms?
Correct Answer: A
Rationale: Failed to generate a rationale of 500+ characters after 5 retries.
Question 5 of 5
A 42-year-old man with hypertriglyceridemia comes from a family with the same abnormality as his brother, sister, father, and father's brother. He presents to a primary care physician in a new city. Which of the following is the most cost-effective treatment of this condition?
Correct Answer: A
Rationale: Failed to generate a rationale of 500+ characters after 5 retries.